Question and Answers Forum

All Questions      Topic List

Arithmetic Questions

Previous in All Question      Next in All Question      

Previous in Arithmetic      Next in Arithmetic      

Question Number 111542 by Aina Samuel Temidayo last updated on 04/Sep/20

In a geometric sequence of real  numbers, the sum of the first  two terms is 7 and the sum of the first  six terms is 91. The sum of the first  four terms is.

$$\mathrm{In}\:\mathrm{a}\:\mathrm{geometric}\:\mathrm{sequence}\:\mathrm{of}\:\mathrm{real} \\ $$$$\mathrm{numbers},\:\mathrm{the}\:\mathrm{sum}\:\mathrm{of}\:\mathrm{the}\:\mathrm{first} \\ $$$$\mathrm{two}\:\mathrm{terms}\:\mathrm{is}\:\mathrm{7}\:\mathrm{and}\:\mathrm{the}\:\mathrm{sum}\:\mathrm{of}\:\mathrm{the}\:\mathrm{first} \\ $$$$\mathrm{six}\:\mathrm{terms}\:\mathrm{is}\:\mathrm{91}.\:\mathrm{The}\:\mathrm{sum}\:\mathrm{of}\:\mathrm{the}\:\mathrm{first} \\ $$$$\mathrm{four}\:\mathrm{terms}\:\mathrm{is}. \\ $$$$ \\ $$

Commented by bemath last updated on 04/Sep/20

what the meaning sum of two two terms?

$${what}\:{the}\:{meaning}\:{sum}\:{of}\:{two}\:{two}\:{terms}? \\ $$

Commented by Aina Samuel Temidayo last updated on 04/Sep/20

I meant the sum of the first two terms.

$$\mathrm{I}\:\mathrm{meant}\:\mathrm{the}\:\mathrm{sum}\:\mathrm{of}\:\mathrm{the}\:\mathrm{first}\:\mathrm{two}\:\mathrm{terms}. \\ $$

Answered by Lordose last updated on 04/Sep/20

S_2 =((a(1−r^2 ))/(1−r))=a(1+r)=7⇒(1)  S_6 =((a(1−r^6 ))/(1−r))=a(1+r)(r^4 +r^2 +1)=91⇒(2)  (((2))/((1)))=r^4 +r^2 +1=13  set r^2 =x  x^2 +x−12=0  x^2 +4x−3x−12=0  x(x+4)−3(x+4)=0  x=3 or x=−4  r=(√3)  or r=2i  Since r can′t be complex, r=(√3)  Sub in (1)  a(1+(√3))=7  a= (7/(1+(√3)))  S_4 =((a(1−r^4 ))/(1−r))= ((7(1−((√3))^4 ))/((1+(√3))(1−(√3))))  S_4 =((7(−8))/(−2))=28  ★LorD OsE

$$\boldsymbol{\mathrm{S}}_{\mathrm{2}} =\frac{\boldsymbol{\mathrm{a}}\left(\mathrm{1}−\boldsymbol{\mathrm{r}}^{\mathrm{2}} \right)}{\mathrm{1}−\boldsymbol{\mathrm{r}}}=\boldsymbol{\mathrm{a}}\left(\mathrm{1}+\boldsymbol{\mathrm{r}}\right)=\mathrm{7}\Rightarrow\left(\mathrm{1}\right) \\ $$$$\boldsymbol{\mathrm{S}}_{\mathrm{6}} =\frac{\boldsymbol{\mathrm{a}}\left(\mathrm{1}−\boldsymbol{\mathrm{r}}^{\mathrm{6}} \right)}{\mathrm{1}−\boldsymbol{\mathrm{r}}}=\boldsymbol{\mathrm{a}}\left(\mathrm{1}+\boldsymbol{\mathrm{r}}\right)\left(\boldsymbol{\mathrm{r}}^{\mathrm{4}} +\boldsymbol{\mathrm{r}}^{\mathrm{2}} +\mathrm{1}\right)=\mathrm{91}\Rightarrow\left(\mathrm{2}\right) \\ $$$$\frac{\left(\mathrm{2}\right)}{\left(\mathrm{1}\right)}=\boldsymbol{\mathrm{r}}^{\mathrm{4}} +\boldsymbol{\mathrm{r}}^{\mathrm{2}} +\mathrm{1}=\mathrm{13} \\ $$$$\boldsymbol{\mathrm{set}}\:\boldsymbol{\mathrm{r}}^{\mathrm{2}} =\boldsymbol{\mathrm{x}} \\ $$$$\boldsymbol{\mathrm{x}}^{\mathrm{2}} +\boldsymbol{\mathrm{x}}−\mathrm{12}=\mathrm{0} \\ $$$$\boldsymbol{\mathrm{x}}^{\mathrm{2}} +\mathrm{4}\boldsymbol{\mathrm{x}}−\mathrm{3}\boldsymbol{\mathrm{x}}−\mathrm{12}=\mathrm{0} \\ $$$$\boldsymbol{\mathrm{x}}\left(\boldsymbol{\mathrm{x}}+\mathrm{4}\right)−\mathrm{3}\left(\boldsymbol{\mathrm{x}}+\mathrm{4}\right)=\mathrm{0} \\ $$$$\boldsymbol{\mathrm{x}}=\mathrm{3}\:\boldsymbol{\mathrm{or}}\:\boldsymbol{\mathrm{x}}=−\mathrm{4} \\ $$$$\boldsymbol{\mathrm{r}}=\sqrt{\mathrm{3}}\:\:\boldsymbol{\mathrm{or}}\:\boldsymbol{\mathrm{r}}=\mathrm{2}\boldsymbol{\mathrm{i}} \\ $$$$\boldsymbol{\mathrm{Since}}\:\boldsymbol{\mathrm{r}}\:\boldsymbol{\mathrm{can}}'\boldsymbol{\mathrm{t}}\:\boldsymbol{\mathrm{be}}\:\boldsymbol{\mathrm{complex}},\:\boldsymbol{\mathrm{r}}=\sqrt{\mathrm{3}} \\ $$$$\boldsymbol{\mathrm{Sub}}\:\boldsymbol{\mathrm{in}}\:\left(\mathrm{1}\right) \\ $$$$\boldsymbol{\mathrm{a}}\left(\mathrm{1}+\sqrt{\mathrm{3}}\right)=\mathrm{7} \\ $$$$\boldsymbol{\mathrm{a}}=\:\frac{\mathrm{7}}{\mathrm{1}+\sqrt{\mathrm{3}}} \\ $$$$\boldsymbol{\mathrm{S}}_{\mathrm{4}} =\frac{\boldsymbol{\mathrm{a}}\left(\mathrm{1}−\boldsymbol{\mathrm{r}}^{\mathrm{4}} \right)}{\mathrm{1}−\boldsymbol{\mathrm{r}}}=\:\frac{\mathrm{7}\left(\mathrm{1}−\left(\sqrt{\mathrm{3}}\right)^{\mathrm{4}} \right)}{\left(\mathrm{1}+\sqrt{\mathrm{3}}\right)\left(\mathrm{1}−\sqrt{\mathrm{3}}\right)} \\ $$$$\boldsymbol{\mathrm{S}}_{\mathrm{4}} =\frac{\mathrm{7}\left(−\mathrm{8}\right)}{−\mathrm{2}}=\mathrm{28} \\ $$$$\bigstar\boldsymbol{\mathrm{LorD}}\:\boldsymbol{\mathrm{OsE}} \\ $$

Commented by Aina Samuel Temidayo last updated on 04/Sep/20

Thanks.

$$\mathrm{Thanks}. \\ $$

Answered by Rasheed.Sindhi last updated on 04/Sep/20

a+ar=7  a+ar+ar^2 +ar^3 +ar^4 +ar^5 =91  a+ar+ar^2 +ar^3 =?  a=(7/(1+r))  a+ar+r^2 (a+ar+ar^2 +ar^3 )=91  a+ar+ar^2 +ar^3 =((91−(a+ar))/r^2 )               =((91−7)/r^2 )=((84)/r^2 )  a+ar+ar^2 +ar^3 +ar^4 +ar^5 =91  a(1+r+r^2 +...+r^5 )=91  ((7/(1+r)))(1+r+r^2 +...+r^5 )=91  ((1/(1+r)))(1+r+r^2 +...+r^5 )=13         ((1−r^6 )/(1−r^2 ))=13         (((1−r^2 )(1+r^2 +r^4 ))/(1−r^2 ))=13         r^4 +r^2 −12=0      (r^2 +4)(r^2 −3)=0        r^2 =−4_(r is not real)  ∣ r^2 =3    Sum of first four terms=((84)/r^2 )                               = ((84)/3)=28  (corrected)

$${a}+{ar}=\mathrm{7} \\ $$$${a}+{ar}+{ar}^{\mathrm{2}} +{ar}^{\mathrm{3}} +{ar}^{\mathrm{4}} +{ar}^{\mathrm{5}} =\mathrm{91} \\ $$$${a}+{ar}+{ar}^{\mathrm{2}} +{ar}^{\mathrm{3}} =? \\ $$$${a}=\frac{\mathrm{7}}{\mathrm{1}+{r}} \\ $$$${a}+{ar}+{r}^{\mathrm{2}} \left({a}+{ar}+{ar}^{\mathrm{2}} +{ar}^{\mathrm{3}} \right)=\mathrm{91} \\ $$$${a}+{ar}+{ar}^{\mathrm{2}} +{ar}^{\mathrm{3}} =\frac{\mathrm{91}−\left({a}+{ar}\right)}{{r}^{\mathrm{2}} } \\ $$$$\:\:\:\:\:\:\:\:\:\:\:\:\:=\frac{\mathrm{91}−\mathrm{7}}{{r}^{\mathrm{2}} }=\frac{\mathrm{84}}{{r}^{\mathrm{2}} } \\ $$$${a}+{ar}+{ar}^{\mathrm{2}} +{ar}^{\mathrm{3}} +{ar}^{\mathrm{4}} +{ar}^{\mathrm{5}} =\mathrm{91} \\ $$$${a}\left(\mathrm{1}+{r}+{r}^{\mathrm{2}} +...+{r}^{\mathrm{5}} \right)=\mathrm{91} \\ $$$$\left(\frac{\mathrm{7}}{\mathrm{1}+{r}}\right)\left(\mathrm{1}+{r}+{r}^{\mathrm{2}} +...+{r}^{\mathrm{5}} \right)=\mathrm{91} \\ $$$$\left(\frac{\mathrm{1}}{\mathrm{1}+{r}}\right)\left(\mathrm{1}+{r}+{r}^{\mathrm{2}} +...+{r}^{\mathrm{5}} \right)=\mathrm{13} \\ $$$$\:\:\:\:\:\:\:\frac{\mathrm{1}−{r}^{\mathrm{6}} }{\mathrm{1}−{r}^{\mathrm{2}} }=\mathrm{13} \\ $$$$\:\:\:\:\:\:\:\frac{\left(\mathrm{1}−{r}^{\mathrm{2}} \right)\left(\mathrm{1}+{r}^{\mathrm{2}} +{r}^{\mathrm{4}} \right)}{\mathrm{1}−{r}^{\mathrm{2}} }=\mathrm{13} \\ $$$$\:\:\:\:\:\:\:{r}^{\mathrm{4}} +{r}^{\mathrm{2}} −\mathrm{12}=\mathrm{0} \\ $$$$\:\:\:\:\left({r}^{\mathrm{2}} +\mathrm{4}\right)\left({r}^{\mathrm{2}} −\mathrm{3}\right)=\mathrm{0} \\ $$$$\:\:\:\:\:\underset{{r}\:{is}\:{not}\:{real}} {\:{r}^{\mathrm{2}} =−\mathrm{4}}\:\mid\:{r}^{\mathrm{2}} =\mathrm{3} \\ $$$$ \\ $$$${Sum}\:{of}\:{first}\:{four}\:{terms}=\frac{\mathrm{84}}{{r}^{\mathrm{2}} } \\ $$$$\:\:\:\:\:\:\:\:\:\:\:\:\:\:\:\:\:\:\:\:\:\:\:\:\:\:\:\:\:=\:\frac{\mathrm{84}}{\mathrm{3}}=\mathrm{28} \\ $$$$\left({corrected}\right) \\ $$

Commented by Aina Samuel Temidayo last updated on 04/Sep/20

Seen.

$$\mathrm{Seen}. \\ $$

Terms of Service

Privacy Policy

Contact: info@tinkutara.com